Có ai đã giải bài tập PTLOS 4.1 chưa?


19

Đây là một bài tập được đưa ra trong Lý thuyết Xác suất: Logic của Khoa học của Edwin Jaynes, 2003. Có một giải pháp một phần ở đây . Tôi đã tìm ra một giải pháp một phần tổng quát hơn, và tự hỏi liệu có ai khác đã giải quyết nó không. Tôi sẽ đợi một chút trước khi đăng câu trả lời của mình, để cho người khác đi.

Được rồi, vì vậy, giả sử chúng ta có giả thuyết loại trừ lẫn nhau và toàn diện, được ký hiệu là . Hơn nữa, giả sử chúng ta có tập dữ liệu, ký hiệu là . Tỷ lệ khả năng cho giả thuyết thứ i được đưa ra bởi:nHi(i=1,,n)mDj(j=1,,m)

LR(Hi)=P(D1D2,Dm|Hi)P(D1D2,Dm|H¯i)

Lưu ý rằng đây là những xác suất có điều kiện. Bây giờ giả sử rằng với giả thuyết thứ i thì các tập dữ liệu là độc lập, vì vậy chúng ta có:Him

P(D1D2,Dm|Hi)=j=1mP(Dj|Hi)(i=1,,n)Condition 1

Bây giờ sẽ khá thuận tiện nếu mẫu số cũng được tính đến trong tình huống này, do đó chúng ta có:

P(D1D2,Dm|H¯i)=j=1mP(Dj|H¯i)(i=1,,n)Condition 2

Trong trường hợp này, tỷ lệ khả năng sẽ chia thành một sản phẩm có các yếu tố nhỏ hơn cho mỗi bộ dữ liệu, do đó chúng tôi có:

LR(Hi)=j=1mP(Dj|Hi)P(Dj|H¯i)

Vì vậy, trong trường hợp này, mỗi bộ dữ liệu sẽ "bỏ phiếu cho Hi " hoặc "bỏ phiếu chống lại Hi " độc lập với bất kỳ bộ dữ liệu nào khác.

Bài tập là để chứng minh rằng nếu n>2 (nhiều hơn hai giả thuyết), không có cách nào không tầm thường như vậy trong đó bao thanh toán này có thể xảy ra. Nghĩa là, nếu bạn giả sử rằng điều kiện 1 và điều kiện 2 giữ, thì nhiều nhất là một trong các yếu tố:

P(D1|Hi)P(D1|H¯i)P(D2|Hi)P(D2|H¯i)P(Dm|Hi)P(Dm|H¯i)
khác với 1 và do đó chỉ có 1 bộ dữ liệu sẽ đóng góp vào tỷ lệ khả năng.

Cá nhân tôi thấy kết quả này khá hấp dẫn, bởi vì về cơ bản nó cho thấy nhiều thử nghiệm giả thuyết không là gì ngoài một loạt các thử nghiệm giả thuyết nhị phân.


Tôi hơi bối rối bởi chỉ mục trên ; là ? Hoặc là ? Có vẻ như nó phải là cái thứ hai, nhưng sau đó tôi không chắc tại sao lại là đăng ký. Hoặc có thể tôi đang thiếu một cái gì đó hoàn toàn khác :) ˉ H i=argmaxhHiP(D1,...Dm|h) ˉ H i=argmaxh{H1,...,Hn}P(D1H¯iH¯i=argmaxhHiP(D1,Dm|h)H¯i=argmaxh{H1,,Hn}P(D1,Dm|h)
JMS

@JMS - là viết tắt của câu lệnh logic " là sai" hoặc một trong những giả thuyết khác là đúng. Vì vậy, trong "đại số Boolean", chúng ta có (vì giả thuyết là độc quyền và toàn diện)Hi ¯ H iH1+H2++Hi-1+Hi+1++HnH¯iHiH¯iH1+H2++Hi1+Hi+1++Hn
xác suất

Tôi cảm thấy như phải có một giải pháp trực quan hơn so với đại số được đưa ra trong giải pháp một phần của Sanders. Nếu dữ liệu là độc lập với từng giả thuyết thì điều này sẽ tiếp tục được giữ khi các linh mục của giả thuyết khác nhau. Và bằng cách nào đó, kết quả là điều tương tự phải áp dụng cho kết luận ...
charles.y.zheng

@ sành - Tôi biết chính xác cảm giác của bạn. Tôi nghĩ rằng tôi có thể lấy được nó bằng cách sử dụng một số mâu thuẫn định tính (Reductio ad absurdum), nhưng tôi không thể làm điều đó. Tôi có thể mở rộng toán học của Sander mặc dù. Và đó là Điều kiện 2 là "sự tinh ranh" về ý nghĩa của kết quả.
xác suất

@probabilityislogic "về cơ bản cho thấy rằng nhiều thử nghiệm giả thuyết không là gì ngoài một loạt các thử nghiệm giả thuyết nhị phân." Xin vui lòng, bạn có thể mở rộng về câu này? Bằng cách đọc trang 98 từ cuốn sách của Jaynes, tôi hiểu rằng bạn có thể giảm việc kiểm tra để kiểm tra đối với giả thuyết của nhau và sau đó bằng cách nào đó bình thường hóa để có được hậu thế cho , nhưng tôi không hiểu tại sao điều này sẽ xảy ra từ kết quả của Excercise 4.1. H 1 H 1H1,,HnH1H1
Martin Drozdik

Câu trả lời:


7

Lý do chúng tôi chấp nhận eq. 4.28 (trong cuốn sách, điều kiện của bạn 1) là chúng tôi giả định xác suất dữ liệu được đưa ra một giả thuyết nhất định và thông tin cơ bản là độc lập, nói cách khác đối với mọi và với : X D i D j i jHaXDiDjij

P(Di|DjHaX)=P(Di|HaX)(1)
Nonextensibility ngoài trường hợp nhị phân do đó có thể được thảo luận như thế này: Nếu chúng ta giả eq .1 là đúng, eq.2 cũng đúng?

P(Di|DjHa¯X)=?P(Di|Ha¯X)(2)
Trước tiên, hãy nhìn vào phía bên trái của eq.2, sử dụng quy tắc nhân:

n { H 1 ... H n } ¯ H một = Σ b một H b P ( D i | D j ¯ H một X ) =

P(Di|DjHa¯X)=P(DiDjHa¯|X)P(DjHa¯|X)(3)
Vì giả thuyết được giả sử loại trừ lẫn nhau và toàn diện, chúng ta có thể viết: Vậy eq.3 trở thành: Đối với trường hợp chúng tôi chỉ có hai giả thuyết, các tổng kết được loại bỏ (vì chỉ có một ), các thuật ngữ bằng nhau trong người đề cử và mẫu số, ), hủy bỏ và eq.2 được chứng minh là đúng, vìn{H1Giáo dụcHn}
Hmột¯= =ΣbmộtHb
baP(DjHb|XHb=¯ H a {H1,H2,H3}P(Di|D)P(DjH2|X)+P(D
P(DTôi|DjHmột¯X)= =ΣbmộtP(DTôi|DjHbX)P(DjHb|X)ΣbmộtP(DjHb|X)= =ΣbmộtP(DTôi|HbX)P(DjHb|X)ΣbmộtP(DjHb|X)
bmộtP(DjHb|XHb= =Hmột¯ . Do đó phương trình 4.29 có thể được bắt nguồn từ phương trình 4.28 trong cuốn sách. Nhưng khi chúng ta có nhiều hơn hai giả thuyết, thì điều này sẽ không xảy ra, ví dụ, nếu chúng ta có ba giả thuyết: , phương trình trên trở thành: Nói cách khác: Cách duy nhất phương trình này có thể mang lại eq.2 là cả hai mẫu số đều bằng 1, tức là cả hai phân số trong mẫu số phải bằng không. Nhưng điều đó là không thể.{H1,H2,H3} P(Di|Dj ¯
P(DTôi|DjH1¯X)= =P(DTôi|H2X)P(DjH2|X)+P(DTôi|H3X)P(DjH3|X)P(DjH2|X)+P(DjH3|X)
P(DTôi|DjH1¯X)= =P(DTôi|H2X)1+P(DjH3|X)P(DjH2|X)+P(DTôi|H3X)1+P(DjH2|X)P(DjH3|X)

1
Tôi nghĩ rằng phương trình thứ tư là không chính xác. Chúng ta nên cóP(DTôiDjHb|X)= =P(DTôiHB|X)P(Dj|HbX)
xác suất

Cảm ơn bạn rất nhiều xác suất, tôi đã có thể sửa giải pháp. Bây giờ bạn đang nghĩ gì?
astroboy

Tôi chỉ không hiểu cách Jaynes nói: "Những người không phân biệt được độc lập logic và độc lập nhân quả sẽ cho rằng (4.29) luôn hợp lệ".
astroboy

Tôi nghĩ rằng tôi đã tìm thấy câu trả lời cho nhận xét cuối cùng của mình: ngay sau câu trên Jaynes nói: "chỉ với điều kiện là không có nào gây ảnh hưởng vật lý lên bất kỳ nào khác ". Vì vậy, về cơ bản, Jaynes đang nói rằng ngay cả khi họ không có ảnh hưởng vật lý, có một giới hạn logic không cho phép khái quát hóa hơn hai giả thuyết. DiDj
astroboy

Sau khi đọc văn bản một lần nữa tôi cảm thấy bình luận cuối cùng của tôi không phải là một câu trả lời tốt. Theo tôi hiểu bây giờ, Jayne muốn nói: "Những người không phân biệt được độc lập logic và độc lập nguyên nhân" sẽ cho rằng và được cho là không có ảnh hưởng vật lý. Do đó, họ có sự độc lập nhân quả mà đối với họ ngụ ý sự độc lập logic đối với bất kỳ giả thuyết nào. Vì vậy, họ thấy tất cả các cuộc thảo luận này là vô nghĩa và chỉ đơn giản là tiến hành khái quát hóa trường hợp nhị phân. D jDTôiDj
astroboy

1

Được rồi, vì vậy thay vì đi và lấy lại phương trình của Saunder (5), tôi sẽ chỉ nêu nó ở đây. Điều kiện 1 và 2 ngụ ý sự bình đẳng sau:

djk=P(Dj|Hk,tôi)

Πj= =1m(ΣkTôihkdjk)= =(ΣkTôihk)m-1(ΣkTôihkΠj= =1mdjk)
trong đó
djk=P(Dj|Hk,I)hk=P(Hk|I)

Bây giờ chúng ta có thể chuyên môn hóa cho trường hợp (hai bộ dữ liệu) bằng cách lấy và nhãn lại . Lưu ý rằng hai bộ dữ liệu này vẫn thỏa mãn điều kiện 1 và 2, do đó, kết quả ở trên cũng áp dụng cho chúng. Bây giờ mở rộng trong trường hợp chúng tôi nhận được:D ( 1 ) 1D 1 D ( 1m=2D1(1)D1 m=2D2(1)D2D3Dmm=2

(kihkd1k)(lihld2l)=(kihk)(lihld1ld2l)

kilihkhld1kd2l=kilihkhld1ld2l

kilihkhld2l(d1kd1l)=0(i=1,,n)

Thuật ngữ xảy ra hai lần trong tổng kết đôi ở trên, một lần khi và , và một lần nữa khi và . Điều này sẽ xảy ra miễn là . Hệ số của mỗi thuật ngữ được cho bởi và . Bây giờ bởi vì có của các phương trình này, chúng tôi thực sự có thể loại bỏ khỏi các phương trình này. Để minh họa, lấy , bây giờ điều này có nghĩa là chúng ta có tất cả các điều kiện ngoại trừ và . Bây giờ lấyk = a l = b k = b l = a a , b i d 2 b - d 2 a i i i = 1(d1ad1b)k=al=bk=bl=aa,bid2b-d2mộtTôiTôiTôi= =1một= =1,b= =2b= =1,một= =2Tôi= =3và bây giờ chúng ta có thể có hai điều kiện này (lưu ý điều này giả sử ít nhất ba giả thuyết). Vì vậy, phương trình có thể được viết lại như sau:

Σtôi>khkhtôi(d2tôi-d2k)(d1k-d1tôi)= =0

Bây giờ mỗi thuật ngữ phải lớn hơn 0, nếu không, chúng tôi đang xử lý giả thuyết và câu trả lời có thể được điều chỉnh lại theo . Vì vậy, những điều này có thể được loại bỏ khỏi các điều kiện trên:hTôin1<nn1

Σtôi>k(d2tôi-d2k)(d1k-d1tôi)= =0

Do đó, có các điều kiện phải được thỏa mãn và mỗi điều kiện ngụ ý một trong hai "điều kiện phụ": đó là cho hoặc (nhưng không nhất thiết cả hai). Bây giờ chúng ta có một tập hợp tất cả các cặp duy nhất cho . Nếu chúng ta lấy của các cặp này cho một trong các , thì chúng ta sẽ có tất cả các số trong tập hợp và . Điều này là do cặp đầu tiên có phần tử và mỗi cặp bổ sung mang lại ít nhất một phần tử bổ sung cho tập hợp *n(n-1)2djk= =djtôij= =1j= =2(k,tôi)djk= =djtôin-1j1,Giáo dục,ndj1=dj2==dj,n1=dj,n2

Nhưng lưu ý rằng vì có các điều kiện , chúng ta phải chọn ít nhất số nguyên nhỏ nhất lớn hơn hoặc bằng cho một trong các hoặc . Nếu thì số lượng thuật ngữ được chọn lớn hơn . Nếu hoặc thì chúng ta phải chọn chính xác điều khoản. Điều này ngụ ý rằng . Chỉ với hai giả thuyết ( ) là nơi điều này không xảy ra. Nhưng từ phương trình cuối cùng trong bài viết của Saunder, điều kiện bình đẳng này ngụ ý:n(n1)212×n(n1)2=n(n1)4j=1j=2n>4n1n=4n=3n1dj1=dj2==dj,n1=dj,nn=2

P(Dj|H¯i)=kidjkhkkihk=djikihkkihk=dji=P(Dj|Hi)

Do đó, trong tỷ lệ khả năng chúng ta có:

P(D1(1)|Hi)P(D1(1)|H¯i)=P(D1|Hi)P(D1|H¯i)=1 ORP(D2(1)|Hi)P(D2(1)|H¯i)=P(D2D3,Dm|Hi)P(D2D3,Dm|H¯i)=1

Để hoàn thành bằng chứng, lưu ý rằng nếu điều kiện thứ hai giữ, kết quả đã được chứng minh và chỉ một tỷ lệ có thể khác với 1. Nếu điều kiện thứ nhất giữ, thì chúng ta có thể lặp lại phân tích ở trên bằng cách lại và . Sau đó, chúng tôi sẽ có không đóng góp hoặc là người đóng góp duy nhất. Sau đó, chúng tôi sẽ có lần dán lại thứ ba khi không đóng góp giữ, v.v. Do đó, chỉ có một bộ dữ liệu có thể đóng góp vào tỷ lệ khả năng khi điều kiện 1 và điều kiện 2 giữ và có nhiều hơn hai giả thuyết.D1(2)D2D2(2)D3,DmD1,D2D2D1D2

* LƯU Ý: Một cặp bổ sung có thể không mang lại điều khoản mới, nhưng điều này sẽ được bù lại bằng một cặp mang lại 2 điều khoản mới. ví dụ: lấy làm đầu tiên [+2], [+1] và [+0], nhưng thuật ngữ tiếp theo phải có cho cả . Điều này sẽ thêm hai thuật ngữ [+2]. Nếu thì chúng ta không cần chọn thêm nữa, nhưng đối với "khác", chúng ta phải chọn 3 cặp không phải là . Đó là và do đó, đẳng thức giữ nguyên, bởi vì tất cả các số đều nằm trong tập hợp.dj1=dj2dj1=dj3dj2=dj3djk=djlk,l(1,2,3)n=4j(1,2),(2,3),(1,3)(1,4),(2,4),(3,4)(1,2,3,4)


Tôi bắt đầu nghi ngờ tính chính xác của bằng chứng này. Kết quả trong toán học Saunders chỉ ngụ ý ràng buộc phi tuyến tính trên . Điều này làm cho chỉ có bậc tự do thay vì . Tuy nhiên, để có được điều kiện cần phải có một đối số khác. ndjkdjkn2nn(n1)2
xác suất

0

Đối với hồ sơ, đây là một bằng chứng rộng rãi hơn . Nó cũng chứa một số thông tin cơ bản. Có lẽ điều này là hữu ích cho những người khác nghiên cứu chủ đề.

Ý tưởng chính của bằng chứng là chỉ ra rằng các điều kiện 1 và 2 của Jaynes ngụ ý rằng cho tất cả trừ một bộ dữ liệu . Sau đó, nó cho thấy rằng đối với tất cả các bộ dữ liệu này, chúng tôi cũng có Do đó, chúng tôi có tất cả trừ một bộ dữ liệu, Lý do tôi muốn đưa ra bằng chứng ở đây là một số bước liên quan hoàn toàn không rõ ràng và người ta cần lưu ý không sử dụng bất cứ thứ gì khác ngoài điều kiện 1 và 2 và quy tắc sản phẩm (như nhiều bằng chứng khác ngầm định làm). Các liên kết ở trên

P(Dmk|HiX)=P(Dmk|X),
mk=1,,m
P(Dmk|H¯iX)=P(Dmk|X).
P(Dmk|HiX)P(Dmk|H¯iX)=P(Dmk|X)P(Dmk|X)=1.
bao gồm tất cả các bước chi tiết. Nó có trên Google Drive của tôi và tôi sẽ đảm bảo nó có thể truy cập được.


Chào mừng bạn đến với Xác thực chéo . Cảm ơn về câu trả lời của bạn. Bạn có thể vui lòng chỉnh sửa câu trả lời của bạn để mở rộng nó, để bao gồm các điểm chính của liên kết bạn cung cấp không? Nó sẽ hữu ích hơn cho cả những người tìm kiếm trong trang web này và trong trường hợp liên kết bị hỏng. Nhân tiện, hãy tận dụng cơ hội để tham quan , nếu bạn chưa thực hiện nó. Xem thêm một số mẹo về Cách trả lời , về định dạng trợ giúp và viết ra các phương trình bằng LaTeX / MathJax .
Ertxiem - phục hồi Monica

Cám ơn bạn đã góp ý. Tôi chỉnh sửa bài và phác thảo các bước chính của bằng chứng.
dennis
Khi sử dụng trang web của chúng tôi, bạn xác nhận rằng bạn đã đọc và hiểu Chính sách cookieChính sách bảo mật của chúng tôi.
Licensed under cc by-sa 3.0 with attribution required.